Quantcast
  • Register
PhysicsOverflow is a next-generation academic platform for physicists and astronomers, including a community peer review system and a postgraduate-level discussion forum analogous to MathOverflow.

Welcome to PhysicsOverflow! PhysicsOverflow is an open platform for community peer review and graduate-level Physics discussion.

Please help promote PhysicsOverflow ads elsewhere if you like it.

News

PO is now at the Physics Department of Bielefeld University!

New printer friendly PO pages!

Migration to Bielefeld University was successful!

Please vote for this year's PhysicsOverflow ads!

Please do help out in categorising submissions. Submit a paper to PhysicsOverflow!

... see more

Tools for paper authors

Submit paper
Claim Paper Authorship

Tools for SE users

Search User
Reclaim SE Account
Request Account Merger
Nativise imported posts
Claim post (deleted users)
Import SE post

Users whose questions have been imported from Physics Stack Exchange, Theoretical Physics Stack Exchange, or any other Stack Exchange site are kindly requested to reclaim their account and not to register as a new user.

Public \(\beta\) tools

Report a bug with a feature
Request a new functionality
404 page design
Send feedback

Attributions

(propose a free ad)

Site Statistics

205 submissions , 163 unreviewed
5,047 questions , 2,200 unanswered
5,345 answers , 22,709 comments
1,470 users with positive rep
816 active unimported users
More ...

  Completing the trace in the Non-abelian Chern-Simons Term

+ 2 like - 0 dislike
651 views

I've been having a little trouble proving that [Page 138 of "Introduction to Topological Quantum Computation" by Jiannis K. Pachos]:

$S_{CS} = \dfrac{k}{4 \pi} \int_{M} d^{3}x \, \epsilon^{\mu \nu \rho} tr \left( A_{\mu}\partial_{\nu}A_{\rho} + i\dfrac{2}{3} A_{\mu} A_{\nu} A_{\rho} \right)$

(with $A_{\mu} = A_{\mu}^{a} T_{a}$ and $T_{a}$ a generator of the gauge group's Lie alegebra) can be written as:

$S_{CS} = \dfrac{k}{8 \pi} \int_{M} d^{3}x \, \epsilon^{\mu \nu \rho} \left( A_{\mu}^{a}\partial_{\nu}A_{\rho}^{a} - \dfrac{1}{3} f_{abc} A_{\mu}^{a} A_{\nu}^{b} A_{\rho}^{c} \right)$

where$f^{abc}$ is antisymmetric.


My attempt thus far:

$tr \left( A_{\mu}\partial_{\nu}A_{\rho} + i\dfrac{2}{3} A_{\mu} A_{\nu} A_{\rho} \right)$

$= tr \left( A_{\mu}^{a} T_{a} \, \partial_{\nu} \, A_{\rho}^{b} T_{b} + i\dfrac{2}{3} A_{\mu}^{a} T_{a} \, A_{\nu}^{b} T_{b} \, A_{\rho}^{c} T_{c} \right)$

$= tr \left( A_{\mu}^{a} \partial_{\nu} A_{\rho}^{b} (T_{a} T_{b}) + i\dfrac{2}{3} A_{\mu}^{a} A_{\nu}^{b} A_{\rho}^{c} (T_{a}T_{b}T_{c}) \right)$

Using $tr(T_{a} T_{b}) = \dfrac{1}{2}\delta_{ab}$ and $(T_{a}T_{b}T_{c}) = \dfrac{1}{2}[T_{a}, T_{b}]T_{c} + \dfrac{1}{2}\{ T_{a}, T_{b} \}T_{c}$ we get:

$= \left( \dfrac{1}{2}A_{\mu}^{a} \partial_{\nu} A_{\rho}^{a} + i\dfrac{1}{3} A_{\mu}^{a} A_{\nu}^{b} A_{\rho}^{c} \, tr \left( [T_{a},T_{b}]T_{c} + \{ T_{a},T_{b} \}T_{c} \right) \right)$

Using

$$[T_{a}, T_{b}] = if_{abd}T_{d}$$

and $\{T_{a}, T_{b}\} = \dfrac{1}{N} \delta_{ab} + d_{abd}T_{d}$, where $d_{abc}$ is symmetric, we get:

$= \left( \dfrac{1}{2}A_{\mu}^{a} \partial_{\nu} A_{\rho}^{a} + i\dfrac{1}{3} A_{\mu}^{a} A_{\nu}^{b} A_{\rho}^{c} \, tr \left( if_{abd}T_{d}T_{c} + \dfrac{1}{N} \delta_{ab} T_{c} + d_{abd}T_{d}T_{c} \right) \right)$

Using $tr(T_{A}) = 0$ and $tr(T_{a} T_{b}) = \dfrac{1}{2}\delta_{ab}$, we get:

$$= \left( \dfrac{1}{2}A_{\mu}^{a} \partial_{\nu} A_{\rho}^{a} + i\dfrac{1}{3} A_{\mu}^{a} A_{\nu}^{b} A_{\rho}^{c} \, \dfrac{1}{2} \left( if_{abc} + d_{abc}\right) \right)$$

Now this would be precisely what we need if the $d_{abc}$ disappeared somehow but I'm not sure how to make that happen. If $A_{\mu}^{a} A_{\nu}^{b} A_{\rho}^{c}$ were anti-symmetric under permutation of the indices (abc) then if would cancel with the symmetric $d_{abc}$ but as far as I'm aware the $A_{\mu}^{a}$'s don't behave that way.

Any help would be greatly appreciated.

Cheers in advance.

This post imported from StackExchange Physics at 2014-06-21 08:58 (UCT), posted by SE-user Siraj R Khan
asked Jun 19, 2014 in Theoretical Physics by Siraj R Khan (105 points) [ no revision ]
It's been a while since I did this calculation but I expect you can use the anti-symmetry of the Levi-Civita symbol $\epsilon^{\mu\nu\rho}$ to show that the unwanted symmetric combination of the gauge fields vanishes.

This post imported from StackExchange Physics at 2014-06-21 08:58 (UCT), posted by SE-user Mark Mitchison
That seems to be it, thank you!

This post imported from StackExchange Physics at 2014-06-21 08:58 (UCT), posted by SE-user Siraj R Khan

1 Answer

+ 6 like - 0 dislike

I think is quite simple if you split the product of two $A_\mu$'s in symmetric and anti-symmetric terms as $$A_\mu A_\nu=\frac{1}{2}[A_\mu,A_\nu]+\frac{1}{2}\{A_{\mu},A_\nu\}\,.$$ Thus, the action of $\epsilon^{\mu\nu\rho}$ on $A_\mu A_\nu$ just kills the symmetric piece $$ \epsilon^{\mu\nu\rho}A_\mu A_\nu=\epsilon^{\mu\nu\rho}\frac{1}{2}[A_\mu,A_\nu]\,. $$ Now, with the normalization $\mathrm{Tr}(T^a T^b)=\delta^{ab}/2$, is trivial to show that $$\mathrm{Tr}\left([A_\mu,A_\nu]A_\rho\right)=A_\mu^a A_\nu^b A^c_\rho \mathrm{Tr}\left([T^a,T^b]T^c\right)=iA_\mu^a A_\nu^b A^c_\rho f^{abd}\mathrm{Tr}(T^d T^c)=i\frac{f^{abc}}{2}A_\mu^a A_\nu^b A^c_\rho$$ and $$\mathrm{Tr}(A_\mu\partial_\nu A_\rho)=\frac{1}{2}A_\mu^a \partial_\nu A^a_\rho$$. Putting everything together we arrive at the result, $$ S_{CS}=\frac{k}{8\pi}\int_M d^3 x\, \epsilon^{\mu\nu\rho}\left(A_\mu^a \partial_\nu A^a_\rho-\frac{1}{3}f^{abc}A_\mu^a A_\nu^b A^c_\rho\right) $$

This post imported from StackExchange Physics at 2014-06-21 08:58 (UCT), posted by SE-user TwoBs
answered Jun 19, 2014 by TwoBs (315 points) [ no revision ]
Thank you for the extremely speedy response, this really helped me out.

This post imported from StackExchange Physics at 2014-06-21 08:58 (UCT), posted by SE-user Siraj R Khan

Your answer

Please use answers only to (at least partly) answer questions. To comment, discuss, or ask for clarification, leave a comment instead.
To mask links under text, please type your text, highlight it, and click the "link" button. You can then enter your link URL.
Please consult the FAQ for as to how to format your post.
This is the answer box; if you want to write a comment instead, please use the 'add comment' button.
Live preview (may slow down editor)   Preview
Your name to display (optional):
Privacy: Your email address will only be used for sending these notifications.
Anti-spam verification:
If you are a human please identify the position of the character covered by the symbol $\varnothing$ in the following word:
$\varnothing\hbar$ysicsOverflow
Then drag the red bullet below over the corresponding character of our banner. When you drop it there, the bullet changes to green (on slow internet connections after a few seconds).
Please complete the anti-spam verification




user contributions licensed under cc by-sa 3.0 with attribution required

Your rights
...